14th USAMO 1985

------
 
 
Problem 5

0 < a1 ≤ a2 ≤ a3 ≤ ... is an unbounded sequence of integers. Let bn = m if am is the first member of the sequence to equal or exceed n. Given that a19 = 85, what is the maximum possible value of a1 + a2 + ... + a19 + b1 + b2 + ... + b85?

 

Solution

We show that the only possible value of the sum is 85.20 = 1700.

That is certainly the value if all ai = 85, for then all bj = 1 and so the sum is 19·85 + 85·1 = 85·20. Now consider the general case. Suppose that we increase some ai by 1 from k to k+1 (whilst preserving the property that ai is increasing, so we must have ai < ai+1 before the increase). The effect of the increase is to change bk+1 from i+1 to i, but not to change any other bj. This is obvious if ai-1 < ai and ai+1 > ai + 1. If ai-1 = ai, then before the change bk = i-1 (not i) and that is still true after the change. Equally, if ai = ai+1 after the change, then it is still true that bk+1 changes from i+1 to i. Thus the overall effect of the increase is not to change the sum of the ai plus the sum of the bj. But by a series of such changes we convert any initial sequence to all ai = 85.

 


 

14th USAMO 1985

© John Scholes
jscholes@kalva.demon.co.uk
26 Aug 2002